Helppp!!! 8 grade!! ​

Helppp!!! 8 Grade!!

Answers

Answer 1

Step-by-step explanation:

for question 1, the change in y is the highest value takeaway the lowest value.

change in y = 10 - 1 = 9

change in x = 6 - (-3) = 9

9/9 = 1

therefore m = 1, this works for all the questions


Related Questions

11. A triangle has a base length of 3.9 cm and a height of 9 cm. Which dimensions could
be the measurements of a similar triangle?

Answers

The dimensions that could be the measurements of a similar triangle are; base = 7.8 cm and height = 18 cm

How to Identify a similar triangle?

Two triangles are said to be similar provided that they have the same ratio of corresponding sides and an equal pair of corresponding angles.

Now, we are given the side lengths of the triangle as a base length of 3.9 cm and a height of 9 cm.

Thus, this means that a similar triangle will have same ratio of side lengths. Thus;

If new height is h and new base is b, then we have;

9/h = 3.9/b

Thus,

h/b = 9/3.9

If we multiply the right side top and bottom by 2, then we have;

h/b = 18/7.8

18 and 7.8 cm could be similar dimensions.

Read more about Similar Triangles at; https://brainly.com/question/14285697

#SPJ1

each year, 40% of a salmon population is extracted from a farming pond. at the beginning of the next year, the pond is stocked with an additional fixed amount of n caught wild salmon. let pn denote the amount of fish at the beginning of the n-th year assume that the initial salmon population on the pond is p0=5000
a. Write a recursion to describe Pn. b. Determine the value of N so the amount of fish remains constant at the beginning of each year.

Answers

(a)     [tex]p_{n} =p_{n-1}-0.4p_{0} +n[/tex]

(b)       [tex]n = {\frac{p_{n-1} -p_{n} }{0.4p_{0}} }[/tex]

Given,

         Initial salmon population [tex]p_{0}=5000[/tex]

         Extraction rate = 40% per year

(a) Formula for recursion,

         For the first year,

                [tex]p_{1} =p_{0} -0.4p_{0}+n[/tex]

         For the second year,

                [tex]p_{2} =p_{1} -0.4p_{0} +n[/tex]

         For the n'th year,

               [tex]p_{n} =p_{n-1}-0.4p_{0} +n[/tex]

(b) The value of n,

        From the above equation,

                [tex]p_{n} =p_{n-1}-0.4p_{0} +n[/tex]

                  [tex]n = {\frac{p_{n-1} -p_{n} }{0.4p_{0}} }[/tex]

Learn more about Recursion here

https://brainly.in/question/634885

#SPJ4

Each year, 40% of the salmon population is extracted from a farming pond.

a. The recursion to describe Pn is Pn = 0.6 Pn-1 + N.

b. The value of N is 3000.

What is recursion?

According to the recursion equation, the quantity of fish at the start of the nth year. Pn, is equal to 0.6 times the quantity at the start of the year before, Pn-1, plus the constant quantity of wild salmon that is restocked every year, N.

b. We can set the recursion equation equal to the initial population of 5000 fish in order to get the value of N that ensures the quantity of fish stays constant at the start of each year. Now we have the equation.

0.6Pn-1 + N = 5000. Solving for N gives us N = 3000.

Therefore, a. Pn = 0.6 Pn-1 + N. b.  3000.

To learn more about recursion, refer to the link:

https://brainly.com/question/16385026

#SPJ1

Given the points (1, -4) and (4, 5), construct two equations in point slope form using each of the points. Construct and upload the graph of the line. Show your work and upload your graph for your teacher to review.

Answers

The equation of line passes through the points (1, -4) and (4, 5) will be;

⇒ y = 3x - 7

What is Equation of line?

The equation of line in point-slope form passing through the points

(x₁ , y₁) and (x₂, y₂) with slope m is defined as;

⇒ y - y₁ = m (x - x₁)

Where, m = (y₂ - y₁) / (x₂ - x₁)

Given that;

Two points on the line are (1, -4) and (4, 5).

Now,

Since, The equation of line passes through the points (1, -4) and (4, 5)

So, We need to find the slope of the line.

Hence, Slope of the line is,

m = (y₂ - y₁) / (x₂ - x₁)

m = (5 - (-4)) / (4 - 1)

m = 9 / 3

m = 3

Thus, The equation of line with slope 3 is,

⇒ y - (-4)= 3 (x - 1)

⇒ y + 4 = 3x - 3

⇒ y = 3x - 3 - 4

⇒ y = 3x - 7

Therefore, The equation of line passes through the points (1, -4) and (4, 5) is given as;

⇒ y = 3x - 7

Learn more about the equation of line visit:

https://brainly.com/question/18831322

#SPJ1

exercise 28 a sample of 20 joint specimens of a particular type gave a sample mean proportional limit stress of 8.52 mpa and a sample standard deviation of 0.78 mpa.

Answers

A 95% % lower confidence bound for the true average proportional limit stress of all such joints is 8.01.

In the given question, a sample of 20 joint specimens of a particular type gave a sample mean proportional limit stress of 8.52 mpa and a sample standard deviation of 0.78 mpa.

We have to calculate and interpret a 95% lower confidence bound for the true average proportional limit stress of all such joints.

Sample size: n = 13

Sample mean: x’ = 8.52 mps

Sample standard deviation: s = 0.78 mps

95% lower confidence bound for the true average proportional limit stress of all such joints

Level of Significance (α) = 1-95% = 5% = 0.05

α/2 = 0.025

So the value of t(α/2) = 1.8

The confidence interval = x’± t(α/2)*s/√n

Now putting the value:

The confidence interval = 8.41± (1.8)*(0.78)/√12

The confidence interval = 8.41± (1.8)*(0.226)

The confidence interval = 8.41± 0.4068

The confidence interval = (8.41- 0.4068, 8.41+ 0.4068)

The confidence interval = (8.01, 8.82)

A 95% % lower confidence bound for the true average proportional limit stress of all such joints is 8.01.

To learn more about confidence interval link is here

brainly.com/question/24131141

#SPJ4

The right question is:

A sample of 12 joint specimens of a particular type gave a sample mean proportional limit stress of 8.41 mpa and a sample standard deviation of 0.78 mpa.

Calculate and interpret a 95% lower confidence bound for the true average proportional limit stress of all such joints. (Round your answer to two decimal places.)

Graph the solution of the inequality on a number line.
2(x − 3) – 5x < x-2
Ο
Α.
OB.
OC
OD.

-5 -4 -3 -2 -1 0 1 2

+1
-5 -4 -3 -2 -1 0 1
#2
0111
1
-5 -4 -3 -2 -1 0
2
3 4 5
3 4 5
3 4
+
5
4
110111
-5 -4 -3 -2 -1 0 1 2 3 4 5

Answers

Answer:

Choice D : x > - 1

Step-by-step explanation:

Left hand side (LHS) of the inequality can be siimplified

[tex]2\left(x-3\right)-5x = 2x - 6 - 5x = -3x -6\\\\[/tex]

Inequality becomes

[tex]-3x-6 < x-2[/tex]

Move x to the left side and -6 to the right side:

[tex]-3x - x < -2 + 6\\\\-4x < 4\\\\[/tex]

Multiply both sides by -1(the inequality sign changes to >)

[tex]4x > -4\\\\[/tex]

Divide both sides by 4:
[tex]x > -1[/tex]

The number line corresponding to this is the fourth choice

he button provides an automatic means of checking for mathematical errors within formulas of a worksheet.

Answers

The Error checking button provides an automatic means of checking for mathematical errors within formulas of a worksheet.

The modeling process begins with the framing of a conceptual model that shows the relationships between the various parts of the problem being modeled.

Arrows pointing from the selected cell to cells that depend on the selected cell are generated by using the Trace dependents button of the Formula Auditing group.

nodes in an influence diagram represent part of the model

The Trace Precedents button, located in the Formula Auditing group, creates arrows pointing to the selected cell from cells that are part of the formula in that cell

The Error checking button provides an automatic means of checking for mathematical errors within formulas of a worksheet.

To know more about error checking button brainly.com/question/1278871

#SPJ4

A laser rangefinder is locked on a comet approaching Earth. The distance g(x) , in kilometers, of the comet after x days, for x in the interval 0 to 42 days, is given by g(x)=300,000csc(π42x) .

Answers

a. The blue line represents g(x) = 250,000csc(π30x).

b. g(5) = 25,000 km. This is the distance of the comet from Earth after 5 days.

c. The minimum distance between the comet and Earth is 250,000 km, which occurs when x = 0. This corresponds to the constant csc(π30x).

d. The equation has a vertical asymptote at x = 30/π.

Write the solution to following questions:a. Graph the value of x on the range [0,35].b. Analyze g(5) and explain the results.c. What is the shortest path the comet must take to reach Earth? When does this take place? What equation constant does this match up to?d. Find any vertical asymptotes and explain their significance.

a. Graph g(x):

To graph g(x), we can plot several points on the interval [0,35] and then connect them to form a graph of the function. For example, we can plot the points (0,250,000), (5,50,000), (10,25,000), (15,16,667), (20,12,500), (25,10,000), (30,8,333) and (35,7,143). Connecting these points will give us a graph of the function g(x).

b. Evaluate g(5):

The value of g(5) can be found by plugging in x = 5 into the equation: g(5) = 250,000 csc(π30*5) = 50,000 km. This means that after 5 days, the comet will be 50,000 km away from Earth.

c. Minimum distance between the comet and Earth:

The minimum distance between the comet and Earth can be found by taking the derivative of the equation and setting it to 0.

d. Vertical Asymptotes:

The vertical asymptotes of g(x) occur at x = 0 and x = 30. This means that the comet approaches an infinitely large distance from Earth at x = 0 and x = 30.

To learn more about asymptotes refer to:

https://brainly.com/question/4138300

#SPJ1

Consider square ABCD. What is the angle of rotation about the center that maps AB to BC?
Recall that rotations are counterclockwise.

Answers

The angle of rotation is 90 degrees.

What is a square?

In a square, all the sides are equal in length and all the angles are right angles (90 degrees). The center of a square is the point that is equidistant from all four vertices, and it is also the point of intersection of the diagonals of the square.

The angle of rotation about the center that maps AB to BC is 90 degrees.

If we rotate the square about its center, the sides of the square will rotate to new positions. For example, if we rotate the square 90 degrees counterclockwise (as specified in the question), side AB will rotate to the position of side BC.

The angle of rotation that maps AB to BC is the angle through which the square was rotated.

Hence, the angle of rotation is 90 degrees.

To learn more about the square, visit:

https://brainly.com/question/25092270

#SPJ1

Write another division problem that has a quotient of 3 and a remainder of 28.

Answers

Division problem with quotient of 3 and a remainder of 28 are-

43 / 157 = 3, leaving a difference of 28.Three times 37 yields three, leaving a leftover of 28.Explain the term quotient of the division?The quotient is the result of dividing two numbers by each other. As in the case of 8 ÷ 4 = 2, when the division produced the number 2, the outcome is the quotient.In this example, the number being divided (15) is known as the dividend, as well as the number being divided by (3 in this instance) is known as the divisor. The quotient is the outcome of the division. Observe how 15 ÷ 3 = 5 is a correct equation even if you change the quotient and divisor. 15 ÷ 5 = 3.

The given data is-

quotient=3remainder=28

To estimate:

division problem?

43 / 157 = 3, leaving a difference of 28.Three times 37 yields three, leaving a leftover of 28.

To know more about the quotient of the division, here

https://brainly.com/question/28867023

#SPJ1

Period:
9) Maria walked 20 miles yesterday. Today she walked 30 miles. What was the percent of increase Maria walked?
10) Pedro goes to 70 % of the basketball teams games. If the basketball team plays 20 games, how many did he go to?
11) Your bill at a restaurant comes to $ 24.00. You want to leave a 15% tip. What is your final bill?
12) Monica wants to buy a bag of Taxis for $13.50. If the store is having a discount of 20% off, how much is she spending?
13) How much interest is earned on $ 100 at 70 % for 9 years?
14) How much interest is earned on a $42 investment at 5 % for three years?

Answers

The percent of increase Maria walked if found as the 50%.

Explain the term percent increase?The amount that a percentage has increased over time is expressed as a percent increase.One would need to figure out the difference between the initial value and the final value, subtract to determine the precise sum of the drop, in order to arrive at this amount.

The formula for the percentage increase is given as;

Percent increase = Increase / original number x 100

original number = 20 miles

Increase = 30 miles - 20 miles

Increase = 10 miles

So,

Percent increase = 10 / 20 x 100

Percent increase = 50%

Thus, the percent of increase Maria walked if found as the 50%.

To know more about the percent increase, here

https://brainly.com/question/11360390

#SPJ1

The correct question is-

9) Maria walked 20 miles yesterday. Today she walked 30 miles. What was the percent of increase Maria walked?

Zareen has 24 minutes to work on her math homework, and each problem is taking her of a minute, on average, to complete. Which expression can be used to determine the number of math problems she will be able to complete in the time she has?

Answers

By using division, it can be concluded that

Number of math problem she can be able to complete within given time is  [tex]24 \div \frac{2}{3}[/tex]

This is the required expression

What is division?

Division is the process by which value of single unit can be calculated from the value of multiple unit.

The number to be divided is known as dividend, the number by which the dividend is divided is the divisor, the result obtained is the quotient and the remaining part is the remainder.

There is a well known formula for division

Divisor x Quotient + Remainder = Dividend.

This is a word problem on division

Zareen has 24 minutes to work on her math homework

Fraction of time taken to do each sum =[tex]\frac{2}{3}[/tex]

Number of math problem she can be able to complete within given time is

[tex]24 \div \frac{2}{3}[/tex]

To learn more about division, refer to the link-

https://brainly.com/question/25289437

#SPJ1

an application is using a two-dimensional list defined as follows: write a statement that creates an empty two-dimensional list named values with 4 rows and 3 columns. write nested loops that get an integer value from the user for each element in the list, example: values

Answers

Complete Question :

An application is using a two-dimensional list defined as follows: 1. Write a statement that creates an empty two-dimensional list named values with 4 rows and 3 columns. 2. Write nested loops that get an integer value from the user for each element in the list, for example: values= [[1, 2, 3], [10, 20, 30], [100, 200, 300],[1000, 2000, 3000]] 3. Write a function named row_values that accepts values as argument and returns the sums of each row and displays the result as a list named row 4. Write a function named column_values that accepts values as arguments and returns the sums of each column and displays the result as a list named column 5. Write a function called sum_values that sums all the elements of the array and displays the result.

Sum of rows : [ 6, 60, 600, 6000 ]

Sum of columns : [ 1111, 2222, 3333]

Sum of all elements : 6666

The nested loops are :

Enter a number at row 0, and column 0 : 1

Enter a number at row 0, and column 1 : 2

Enter a number at row 0, and column 2 : 3

Enter a number at row 1, and column 0 : 10

Enter a number at row 1, and column 1 : 20

Enter a number at row 1, and column 2 : 30

Enter a number at row 2, and column 0 : 100

Enter a number at row 2, and column 1 : 200

Enter a number at row 2, and column 2: 300

Enter a number at row 3, and column 0 : 1000

Enter a number at row 3, and column 1 : 2000

Enter a number at row 3, and column 2 : 3000

To read more about nested loops, visit https://brainly.com/question/29973787

#SPJ4


What is the possible value of a positive, even integer that's shaped like a 4?

Answers

Answer: It's not possible for a positive, even integer to have a specific shape, such as the shape of a 4. This is because integers are abstract mathematical concepts that represent whole numbers and do not have any physical properties, such as shape. Additionally, even integers are simply integers that are divisible by 2, and do not have any specific shape associated with them.

If you're asking about a specific number that has the shape of a 4, such as the number "4" written in a specific way, it's important to note that the value of a number does not depend on its physical appearance. For example, the number 4 written as "4" and the number 4 written as "IV" (the Roman numeral for 4) are both the same number, and have the same value.

Evaluate the expression for the given value of the variable 4c - 3. C= -2

Answers

Answer:

-11

Step-by-step explanation:

4c - 3   Substitute -2 for c

4(-2) - 3

-8 - 3

-11

Answer:

-11

Step-by-step explanation:

We are given the value of c, so we just need to plug it in. We can do this by replacing "c" with "-2" in our given expression.

4c - 3 ⇒ 4(-2) - 3

Now, we can simplify this.

-8 - 3 = -11

Learn more about evaluating expressions by reviewing these answers!

⊕ https://brainly.com/question/13322249

⊕ https://brainly.com/question/16980257

What is the area of this triangle?
Enter your answer in the box.
units?

Answers

Answer:

Below

Step-by-step explanation:

If you turn it sideways and use the y-axis portion as the base (4)  , you can see height = 5

Area = 1/2 * b * h =    1/2 *  4 *5 = 10 units^2

Write the expression \[\frac{4+6a}{5}-\frac{1+3a}{4}\] as a single fraction.

Answers

The expression [tex]\[\frac{4+6a}{5}-\frac{1+3a}{4}\][/tex] as a single fraction is [tex]\frac{9a-11}{20}[/tex]

What is algebraic expression ?

In mathematics, an expression that incorporates variables, constants, and algebraic operations is known as an algebraic expression (addition, subtraction, etc.). Terms comprise expressions.

According to question

⇒     [tex]\[\frac{4+6a}{5}-\frac{1+3a}{4}\][/tex]

⇒   taking LCM of 4, 5 = 20

now [tex]\[\frac{4(4+6a)-5(1+3a)}{20}[/tex]

⇒ (16 +24a - 5 - 15a)/20

⇒ ((24a - 15a) - (16-5))/20

⇒ (9a - 11)/20

hence, the expression  [tex]\[\frac{4+6a}{5}-\frac{1+3a}{4}\][/tex] as a single fraction is [tex]\frac{9a-11}{20}[/tex]

To learn more about  Algebraic expression  , check out

https://brainly.com/question/953809

#SPJ1

A graphing calculator is recommended.
Three students, Linda, Tuan, and Javier, are given five laboratory rats each for a nutritional experiment. Each rat's weight is recorded in grams. Linda feeds her rats Formula A, Tuan feeds his rats Formula B, and Javier feeds his rats Formula C. At the end of a specified time period, each rat is weighed again, and the net gain in grams is recorded. Using a significance level of 10%, test the hypothesis that the three formulas produce the same mean weight gain. (Let 1 = Linda's rats, 2 = Tuan's rats and 3 = Javier's rats.)
Weights of Student Lab Rats
Linda's rats Tuan's rats Javier's rats
46.3 49.8 53.3
42.3 42.7 43.1
44.1 41.8 40.2
48.7 48.9 47.7
40.8 46.5 51.5Enter an exact number as an integer, fraction, or decimal.
df(num) =
Enter an exact number as an integer, fraction, or decimal.
df(denom) =
State the distribution to use for the test.
A. F2, 12
B. F12, 2
C. F14, 2
D. F14, 12
E. F2, 14
What is the test statistic? (Round your answer to two decimal places.)
What is the p-value? (Round your answer to four decimal places.)
Explain what the p-value means for this problem.
A. If H0 is false, then there is a chance equal to the p-value that the value of the test statistic will be equal to or less than the calculated value.
B. If H0 is true, then there is a chance equal to the p-value that the value of the test statistic will be equal to or less than the calculated value.
C. If H0 is false, then there is a chance equal to the p-value that the value of the test statistic will be equal to or greater than the calculated value.
D. If H0 is true, then there is a chance equal to the p-value that the value of the test statistic will be equal to or greater than the calculated value.
Indicate the correct decision ("reject" or "do not reject" the null hypothesis), the reason for it, and write appropriate conclusions.
(i) Alpha (Enter an exact number as an integer, fraction, or decimal.)
α =
(ii) Decision:
reject the null hypothesisdo not reject the null hypothesis
(iii) Reason for decision:
Since α < p-value, we do not reject the null hypothesis.
Since α > p-value, we reject the null hypothesis.
Since α < p-value, we reject the null hypothesis.
Since α > p-value, we do not reject the null hypothesis.
(iv) Conclusion:
There is sufficient evidence to conclude that there is a difference among the different nutritional formulas for rats with respect to weight gain.
There is not sufficient evidence to conclude that there is a difference among the different nutritional formulas for rats with respect to weight gain.

Answers

The graph representation of the given distribution is attached below.

The term graph in math refers the visual representation of the collection of data that has the x and y coordinate values.

Here we have given that Three students, Linda, Tuan, and Javier, are given five laboratory rats each for a nutritional experiment. Each rat's weight is recorded in grams and we need to find the graphical representation of the distribution.

Here by using the values in the foregoing ANOVA table are quickly produced by the calculator, including the test statistic and the p-value of the test calculator shows :

=> F=0.66853555

=> p = 0.530548

And the FACTOR df =2 and SS=23.212

=> MS = 11.606

Here  the ERROR df = 12

=> SS=208.324

=>  MS = 17.36

Here by using the f distribution for the test of three different means.

Then the value of the test statistic (F-value) is 0.668 and the p -value for the test is 0.5301

When we plot the graph for the distribution then we get he graph like the following.

To know more about Graph here.

https://brainly.com/question/17267403

#SPJ4

what is 309x317/9-4+7/8x3

Answers

Answer:

[tex]340.11[/tex] or [tex]\frac{32651}{96}[/tex].

Step-by-step explanation:

1. Write the expression.
[tex]\frac{\frac{309*317}{9-4+7}}{8x3}[/tex]

2. Add parentheses to the expression.

[tex]\frac{(\frac{309*317}{9-4+7})}{8x3}[/tex]

3. Solve the main denomonator.

[tex]\frac{(\frac{309*317}{9-4+7})}{24}[/tex]

4. Solve the secondary denominator.

[tex]\frac{(\frac{309*317}{12})}{24}[/tex]

5. Solve the secondary numerator.

[tex]\frac{(\frac{97953}{12})}{24}[/tex]

6. Solve the main numerator.

[tex]\frac{(8162.75)}{24}[/tex]

7. Simplify the fraction.

[tex]340.11[/tex] or [tex]\frac{32651}{96}[/tex].

What is the product?

2•[-1]
[0]

Answers

Answer:

-2

Step-by-step explanation:

If a loading ramp is placed next to a truck, at a height of 8 feet, and the inclined portion of the ramp is 23 feet long, what angle (in degrees) does the ramp make with the ground?

Answers

Answer:

≈20.35°

Step-by-step explanation:

height = 8

hypotenuse =23

angle is x

sin(x) =8/23

[tex]sin^{-1} (\frac{8}{23} )[/tex] ≈20.35°

x is about ≈20.35°

The ramp makes an angle of approximately 20.86 degrees with the ground.

Given that there is a truck with a ramp at a height of 8 feet the size of the ramp is 23 feet,

we need to find the angle ramp make with the ground.

To find the angle that the ramp makes with the ground, we can use the sine function.

The sine of an angle is equal to the opposite side divided by the hypotenuse in a right triangle.

In this case, the opposite side is the height of the ramp (8 feet), and the hypotenuse is the length of the inclined portion of the ramp (23 feet).

Let's calculate the angle:

sin(angle) = opposite/hypotenuse

sin(angle) = 8/23

To find the angle, we need to take the inverse sine (or arcsine) of both sides of the equation:

angle = arcsin(8/23)

We can find the approximate value of the angle:

angle ≈ 20.86 degrees

Therefore, the ramp makes an angle of approximately 20.86 degrees with the ground.

Learn more about sine function click;

https://brainly.com/question/23556818

#SPJ2

A firm manufactures two products; the net profit on product 1 is Rupees 3 per

unit and Rupees 5 per unit on product 2. The manufacturing process is such that

each product has to be processed in two departments D1 and D2. Each unit of

product1 requires processing for 1 minute at D1 and 3 minutes at D2; each unit

of product 2 requires processing for 2 minutes at D1 and 2 minutes at D2.

Machine time available per day is 860 minutes at D1 and 1200 minutes at D2.

How much of product 1 and 2 should be produced every day so that total profit

is maximum. Make the mathematical model for the given problem.


Answers

To answer this question we need to make use of Linear Programming

The solution is:

x = 170 units

y = 345 units

z(max) = 2235 rupees

To solve a linear programming problem, we need to formulate the model  

The Objective Function

Let´s call x the number of product 1 manufactured

and y the number of product 2 manufactured

Then the Objective Function is:

z = 3× x + 5×y    to be maximize

The set of constraints are:

                             D1           D2

                           ( min. )     ( min. )

Product  1 (x)          1               3

Product 2 (y)          2              2            

Availability            860        1200

First constraint:

Time available in D1:  860 minutes

1×x  +  2×y  ≤ 860

Second constraint:

Time available in D2: 1200 minutes

3×x  +  2×y  ≤ 1200

General constraint:   x ≥ 0  ;  y ≥ 0   integers ( we will assume only complete products at the end of the period no fractions )

Then the model is:

z = 3× x + 5×y    to be maximize

Subject to:  

1×x  +  2×y  ≤ 860

3×x  +  2×y  ≤ 1200

x ≥0   y ≥ 0   integers

With the help of Avtomat we get the solution:

x = 170 units

y = 345 units

z(max) = 2235 rupees

To learn more about Programming visit

https://brainly.com/question/11023419

#SPJ1

evaluate the six trigonometric functions at 11pi/3. (give an exact answer, not a decimal approximation.)

Answers

The six trigonometric functions at 11π/3 are as follows-

sin (11π/3) = -[tex]\sqrt{3}[/tex]/2

cos (11π/3) = 1/2

tan (11π/3) = -[tex]\sqrt{3}[/tex]

cot (11π/3) = -1/[tex]\sqrt{3}[/tex]

sec (11π/3) = 2

cosec (11π/3) = -2/ [tex]\sqrt{3}[/tex]

Let us evaluate the six trigonometric functions at 11π/3 as follows -

11π/3 can be written as (6π+5π)/3 = 2π + 5π/3.

a. sin (11π/3) = sin (2π + 5π/3)

this angle is greater than or equal to 0 and less than 2π.

= sin (5π/3)

Finding the angle with equivalent trigonometric values in the first quadrant. Also, sin is negative in the 4th quadrant.

= -sin (π/3)

= -[tex]\sqrt{3}[/tex]/2

Hence, sin (11π/3) = -[tex]\sqrt{3}[/tex]/2

b. cos (11π/3) = cos (2π + 5π/3)

this angle is greater than or equal to 0 and less than 2π.

= cos (5π/3)

Finding the angle with equivalent trigonometric values in the first quadrant. Also, cos is positive in the 4th quadrant.

= cos (π/3)

= 1/2

Hence, cos (11π/3) = 1/2

c. tan (11π/3) = tan (2π + 5π/3)

this angle is greater than or equal to 0 and less than 2π.

= tan (5π/3)

Finding the angle with equivalent trigonometric values in the first quadrant. Also, tan is negative in the 4th quadrant.

= -tan (π/3)

= -[tex]\sqrt{3}[/tex]

Hence, tan (11π/3) = -[tex]\sqrt{3}[/tex]

d. cot (11π/3) = 1/ tan (11π/3)

Hence, cot (11π/3) = - 1/ [tex]\sqrt{3}[/tex]

e. sec (11π/3) = 1/ cos (11π/3)

Hence, sec (11π/3) = 2

f. cosec (11π/3) = 1/ sin (11π/3)

Hence, cosec (11π/3) = -2/ [tex]\sqrt{3}[/tex]

Read more about trigonometric functions:

brainly.com/question/24349828

#SPJ4

Solve the system if we x-3y=0 and 3x-6y=9 by combining the equations.

Answers

Step-by-step explanation:

Hope u get it x=3×3

x=9

It didn't include in the photo

The Picture and Sound electronics store has hired Brennan to work in the warehouse. He uses a pulley with a hand crank to lift heavy boxes. He turns the crank 15 times to lift a box 15 feet. He turns the crank 45 times to lift a box 45 feet.
In this relationship, x represents the number of times Brennan turns the crank to lift a box, and y represents the height the box has been lifted (in feet).
Graph two points for this relationship and the line passing through them.

Answers

The graph (created with MS Excel) of the height the box is lifted to the number of times Brennan turns the crank to lift the box which is the graph of the proportional relationship, y = x, is attached.

What is a proportional relationship?

A proportional relationship is one in which one variable (the output variable) is a constant multiple of the another variable known as an input variable.

The number of times Brennan turns the crank = x

The height to which the box is lifted = y

The points on the graph are; (15, 15), and (45, 45)

The ratio of the y-values to the x-values is 15/15 = 45/45 = 1, which indicates that the relationship is a proportional relationship

A point on the graph of a proportional relationship is the point (0, 0), therefore, the points on the graph are; (0, 0), (15, 15), and (45, 45)

Please find attached the graph of the proportional relationship between the number of times Brennan turns the crank to the height to which the box is lifted created with MS Excel

The relationship between the variables x and y is; y = x

Learn more about proportional relationships here:

https://brainly.com/question/19752715

#SPJ1

Graph the rational function Start by drawing the vertical and horizontal asymptotes. Then plot two points on each piece of the graph. Finally, click on the graph-a-function button X ?

Answers

The horizontal and vertical asymptotes for the function f(x) =6/(-2x +1) is equal to y = 0 and x = 1/2 respectively.

Graph is attached.

As given in the question,

Given function is :

f(x) =6 /(- 2x +1)

f(x) = y

Degree of the numerator is zero

Degree of the denominator is 1.

Degree of denominator is greater than the degree of numerator.

Function represent the horizontal asymptotes for y =0

To get the vertical asymptotes equate denominator equals to zero

(- 2x +1) = 0

⇒2x = 1

⇒x = 1/2

Vertical asymptotes is given by x =1/2.

Graph is attached.

Therefore , the horizontal and vertical asymptotes for the given function is equal to y=0 and x =1/2 respectively.

The above question is incomplete, the complete question is:

Graph the rational function. f(x) =6 /(- 2x +1) Start by drawing the vertical and horizontal asymptotes. Then plot two points on each piece of the graph. Finally, click on the graph-a-function button X.

Learn more about asymptotes here

brainly.com/question/4084552

#SPJ4

NO LINKS!! Please help me with this problem. Part 1gg​

Answers

Answer:

Graph A

Step-by-step explanation:

Given piecewise function:

[tex]f(x)=\begin{cases}9+x, \;\;\:\:x\leq3\\x^2+3,\;\;x > 3\end{cases}[/tex]

To graph the given piecewise function:

Draw the line  y = 9 + x  for the domain (-∞, 3].Draw the curve  y = x² + 3  for the domain (3, ∞).

The line  y = 9 + x  is a straight line with a positive gradient.  

It crosses the x-axis at (-9, 0) and crosses the y-axis at (0, 9).

Therefore, the only graph that shows the line  y = 9 + x  is the first graph.

A reactor coolant tank is being filled with coolant at the rate of 400 gallons per hour with t>0 measure in hours. If the tank originally contained 200 gallons of coolant, approximately how many gallons are in the tank after 8 hours?

Answers

The gallons after 8 hours will be 3400 gallons.

How to illustrate the expression?

Expression refers to the mathematical statements which have at least two terms which are related by an operator and contain either numbers, variables, or both. Addition, subtraction, multiplication, and division are all possible mathematical operations. As an illustration, the expression x + y has the terms x and y with an addition operator between them.

Numbers, variables, operations, functions, brackets, punctuation, and grouping can all be represented by mathematical symbols, which can also be used to indicate the logical syntax's order of operations.

In this case, the reactor coolant tank is being filled with coolant at the rate of 400 gallons per hour with t>0 measure in hours.

Therefore, the gallons after 8 hours will be:

= 200 + 400(8)

= 200 + 3200

= 3400 gallons.

Learn more about expressions on:

brainly.com/question/723406

#SPJ1

Helppp pleaseee it’s due today pleasee

Answers

Answer:

4

6

y=4x+6

Step-by-step explanation:

check the attached file

PLEASE ANSWERE ASAP

If BC = 16.7 ft, what is AY?

Answers

The length of segment AY is 16.7 ft if the length of segment BC is 16.7 ft

How to determine the length of the side length AY

From the question, we have the following parameters that can be used in our computation:

BC = 16.7 ft

The measures of the angles are not given

However, the marks on the side lengths imply that:

The points A, B and C divides the line segments XY, XZ and YZ into equal segmentsThe parallel segments are equal segments

Using the above as a guide, we have the following:

AY = BC

This is because these segments are parallel segments

Recall that parallel segments are equal segments

Substitute the known values in the above equation, so, we have the following representation

AY = 16.7

Hence, the length of AY is 16.7 ft

Read more about triangles at

brainly.com/question/14285697

#SPJ1

Can anyone help me ASAP?

Answers

A cheeseburger costs 4 and a taco costs 2, between the friends one bought one mroe cheeseburger and that upped the price by $4 so if you take 4 away from 12 you’ll get 8 and divide that by 4 and you’ll get 2 (the costs of 1 taco)

Answer:

Equation 1 - x + 4y = 12

Equation 2 -  2x + 4x = 16

Cost of 1 Cheeseburger - $4

Cost of 1 Taco - $2

Step-by-step explanation:

Cheeseburger(x) and Taco(y)

x + 4y = 12 and 2x + 4y = 16

x = - 4y + 12

2(- 4y + 12) + 4y = 16

- 8y + 24 + 4y = 16

- 4y = - 8

y = 2

x + 4(2) = 12

x + 8 = 12

x = 4

Other Questions
How do scientists study the interactions and flow of matter and energy within and between earths Determine all cache accesses and the state of the data cache at any given time. From this you can determine any additional cycles from data cache misses. You must calculate the number of bits for the offset, index, and tag, determine the number of hits and misses, show the contents of the cache, and use the number of misses to determine any additional clock cycles required. The total clock cycles would be the total cycles for all iterations from Part 2 plus any additional memory access cycles from cache misses from Part 3. a nurse is preparing to suction a patient. the pulse is 65 and pulse oximetry is 94%. which finding will cause the nurse to stop suctioning? in which of the following theoretical statements could we be sure that reverse causality would not be problematic? The decomposition of a single compound at 349 K has a rate constant of 4.10 x 10-3 M s1. If the initialconcentration of the reactant is 1.304 M, what is the concentration of the reactant after 90.45 seconds?(the answer should be entered with 3 significant figures; do not enter units; give answer in normalnotation-examples include 1.23 and 12.3 and 120. and -123)Selected Answer:Correct Answer:0.9330.933 1% usage rate segmentation best explains how a fraction of the segment can make up a portion of consumption. suppose hondurans began purchasing real assets in the united states. how would this impact the foreign exchange market for the lempira and the dollar price of the lempira? demand for lempira / dollar price of lempira Which of the following statements about the opportunity cost associated with a capital budgeting project is correct?A project's opportunity cost is a cash outlay that the firm has already paid; therefore, it should not be included in a capital budgeting analysis.The terms sunk cost and opportunity cost generally are used interchangeably.A project's opportunity cost is the return (cash flow) that will not be earned (generated) if funds are invested in a particular capital budgeting project.A project's opportunity cost is not a relevant cash flow, therefore it should not be included in the capital budgeting analysis. An online furniture store sells chairs for $100 each and tables for $400 each. Everyday, the store can ship no more than 18 pieces of furniture and must sell no less than$2900 worth of chairs and tables. Also, the store must sell a minimum of 10 tables. Ifa represents the number of tables sold and y represents the number of chairs sold,write and solve a system of inequalities graphically and determine one possiblesolution.PLS INCLUDE THE INEQUALITIES What is the ke of an object that has a mass of 2kg and a velocity of 8m/s pls help due asap!!!!!!!!!!! Write an equation of the line that passes through the points.4.(-3,0), (-2, 3)5. (-6, 10), (6, -10) One end of an object has a definite positive charge, which is an instance of _____ or a positive or negative state that reacts to an electric field. the nurse responds to the call light of a client who has had a cervical discectomy earlier in the day. the client states that she is having severe pain that had a sudden onset. what is the nurse's most appropriate action? market structures for each of the following scenarios, identify the number of firms present, the type of product, and the appropriate market model. select the matching entry for each dropdown box in the following table. scenario number of firms type of product market model in a major metropolitan area, there are many coffee shops, but one chain has gained a large market share because customers feel its coffee tastes better than its competitors'. there are dozens of pasta producers that sell pasta to hundreds of italian restaurants nationwide. the restaurant owners buy from the cheapest pasta producer they can. while pasta manufacturers must pay licensing fees to their local government and undergo regular food-safety inspections, anyone who has passed inspections can acquire and maintain their license. many identical perfect competition in a small town, there are four providers of broadband internet access: a cable company, the phone company, and two satellite companies. the internet access offered by all four providers is of the same speed. almost everyone in the city already has broadband, so any potential new company would have to engage in a price war with the existing companies and would be unlikely to cover its costs for years, if ever. the government has granted the u.s. postal service the exclusive right to deliver mail. Hooke's law states that the restoring force of a spring is directly proportional to a small displacement. Students in physics class were conducting an experiment to determine the difference in displacement among various springs. They wanted to determine if displacement not only varied with mass but also with the type of spring used. The class tested twenty springs; they varied in length, circumference, and material. They used an assortment of masses as well. On some springs they used masses of 5g increments; on others, masses of 10g increments. After hanging the masses from each spring, they measured how far the springs stretched. The students concluded that thin, aluminum springs had greater displacement than short, thick, iron springs. How could the students improve the validity of the data in this experiment to better support their conclusion?. scientists understand that viruses have advanced greatly in the last 50 years. which statement gives evidence that a virus is non living. Which of these improvements was brought about during the Gilded Age?O Decreased criminal activityO Equal educational opportunitiesReligious freedomSettlement houses give the number of sigma bonds and pi bonds for the molecule below: 3 sigma bonds, 6 pi bonds 6 sigma bonds, 3 pi bonds 6 sigma bonds, 6 pi bonds 12 sigma bonds, 3 pi bonds If you are holding a bucket steady by a rope with a tension force of 20 N, how much does the bucket weigh?